2018 AMC 12B Problem #7

Algebra Level 3

What is the value of

log 3 7 log 5 9 log 7 11 log 9 13 log 21 25 log 23 27 ? \log_3 7 \log_5 9 \log_7 11 \log_9 13 \cdots \log_{21} 25 \log_{23} 27?


This problem is part of the 2018 AMC 12B .
3 log 7 23 3 \log_7 23 9 6 10 3

This section requires Javascript.
You are seeing this because something didn't load right. We suggest you, (a) try refreshing the page, (b) enabling javascript if it is disabled on your browser and, finally, (c) loading the non-javascript version of this page . We're sorry about the hassle.

2 solutions

Chew-Seong Cheong
Feb 17, 2018

P = log 3 7 log 5 9 log 7 11 log 9 13 log 21 25 log 23 27 = log 7 log 9 log 11 log 13 log 25 log 27 log 3 log 5 log 7 log 9 log 21 log 23 = log 7 log 9 log 11 log 13 log 25 log 27 log 3 log 5 log 7 log 9 log 21 log 23 = log 25 log 27 log 3 log 5 = 2 log 5 × 3 log 3 log 3 log 5 = 6 \begin{aligned} P & = \log_3 7\log_5 9 \log_7 11 \log_9 13 \cdots \log_{21} 25 \log_{23} 27 \\ & = \frac {\log 7 \log 9 \log 11 \log 13 \cdots \log 25 \log 27} {\log 3 \log 5 \log 7 \log 9 \cdots \log 21 \log 23} \\ & = \frac {\cancel{\log 7} \cancel{\log 9} \cancel{\log 11} \cancel{\log 13} \cdots \log 25 \log 27}{\log 3 \log 5 \cancel{\log 7} \cancel{\log 9} \cdots \cancel{\log 21} \cancel{\log 23}} \\ & = \frac {\log 25 \log 27}{\log 3 \log 5} \\ & = \frac {2 \cancel{\log 5} \times 3 \cancel{\log 3}}{\cancel{\log 3} \cancel{\log 5}} \\ & = \boxed{6} \end{aligned}

Marta Reece
Feb 16, 2018

log 3 7 log 5 9 log 7 11 log 9 13 log 21 25 log 23 27 = \log_3 7 \cdot \log_5 9 \cdot \log_7 11 \cdot \log_9 13 \cdots \log_{21} 25 \cdot \log_{23} 27=

l n 7 l n 3 l n 9 l n 5 l n 11 l n 7 l n 13 l n 9 l n 25 l n 21 l n 27 l n 23 = \frac{ln 7}{ln 3}\cdot\frac{ln 9}{ln 5}\cdot\frac{ln 11}{ln 7}\cdot\frac{ln 13}{ln 9}\cdots\frac{ln 25}{ln 21}\cdot\frac{ln 27}{ln 23}=

1 l n 3 1 l n 5 l n 25 1 l n 27 1 = l n 27 l n 3 l n 25 l n 5 = 3 l n 3 l n 3 2 l n 5 l n 5 = 3 2 = 6 \frac1{ln 3}\cdot\frac1{ln 5}\cdot\frac{ln 25}1\cdot\frac{ln 27}1=\frac{ln 27}{ln 3}\cdot\frac{ln 25}{ln 5}=\frac{3\cdot ln 3}{ln 3}\cdot\frac{2\cdot ln 5}{ln 5}=3\cdot2=\boxed6

0 pending reports

×

Problem Loading...

Note Loading...

Set Loading...